Convex function exercise [closed]












-1












$begingroup$


having a bit of trouble with this exercise;



Let f be a function, convex on $mathbb{I}$, by writing $sum_{k=1}^{n}x_klambda_k $ for $ n ge 3 $ under the form $x_nlambda_n+(1-lambda_n)y_n$ with $y_n$ expressed with $x_k, kin [|1,n|] $ and $ lambda_q, qin[|1,n|]$ and $lambda_iin]0,1[$and $sum_{n}^{i=1}lambda_i = 1$ .
Prove that :
$sum_{k=1}^{n}lambda_kf(x_k)ge f(sum_{k=1}^{n}x_klambda_k)$.



The previous question was to demonstrate that ;
$zin ]x,y[ <=> existslambda, z=xlambda + (1-lambda)y$










share|cite|improve this question









$endgroup$



closed as off-topic by amWhy, Paul Frost, Eevee Trainer, Saad, Lord_Farin Dec 26 '18 at 9:33


This question appears to be off-topic. The users who voted to close gave this specific reason:


  • "This question is missing context or other details: Please provide additional context, which ideally explains why the question is relevant to you and our community. Some forms of context include: background and motivation, relevant definitions, source, possible strategies, your current progress, why the question is interesting or important, etc." – amWhy, Paul Frost, Eevee Trainer, Saad, Lord_Farin

If this question can be reworded to fit the rules in the help center, please edit the question.
















  • $begingroup$
    May be n dimensional Jensen's inequality may fit the bill?
    $endgroup$
    – Satish Ramanathan
    Dec 25 '18 at 17:26






  • 1




    $begingroup$
    This needs rewriting. It is strangely confusing.
    $endgroup$
    – zhw.
    Dec 25 '18 at 17:59
















-1












$begingroup$


having a bit of trouble with this exercise;



Let f be a function, convex on $mathbb{I}$, by writing $sum_{k=1}^{n}x_klambda_k $ for $ n ge 3 $ under the form $x_nlambda_n+(1-lambda_n)y_n$ with $y_n$ expressed with $x_k, kin [|1,n|] $ and $ lambda_q, qin[|1,n|]$ and $lambda_iin]0,1[$and $sum_{n}^{i=1}lambda_i = 1$ .
Prove that :
$sum_{k=1}^{n}lambda_kf(x_k)ge f(sum_{k=1}^{n}x_klambda_k)$.



The previous question was to demonstrate that ;
$zin ]x,y[ <=> existslambda, z=xlambda + (1-lambda)y$










share|cite|improve this question









$endgroup$



closed as off-topic by amWhy, Paul Frost, Eevee Trainer, Saad, Lord_Farin Dec 26 '18 at 9:33


This question appears to be off-topic. The users who voted to close gave this specific reason:


  • "This question is missing context or other details: Please provide additional context, which ideally explains why the question is relevant to you and our community. Some forms of context include: background and motivation, relevant definitions, source, possible strategies, your current progress, why the question is interesting or important, etc." – amWhy, Paul Frost, Eevee Trainer, Saad, Lord_Farin

If this question can be reworded to fit the rules in the help center, please edit the question.
















  • $begingroup$
    May be n dimensional Jensen's inequality may fit the bill?
    $endgroup$
    – Satish Ramanathan
    Dec 25 '18 at 17:26






  • 1




    $begingroup$
    This needs rewriting. It is strangely confusing.
    $endgroup$
    – zhw.
    Dec 25 '18 at 17:59














-1












-1








-1





$begingroup$


having a bit of trouble with this exercise;



Let f be a function, convex on $mathbb{I}$, by writing $sum_{k=1}^{n}x_klambda_k $ for $ n ge 3 $ under the form $x_nlambda_n+(1-lambda_n)y_n$ with $y_n$ expressed with $x_k, kin [|1,n|] $ and $ lambda_q, qin[|1,n|]$ and $lambda_iin]0,1[$and $sum_{n}^{i=1}lambda_i = 1$ .
Prove that :
$sum_{k=1}^{n}lambda_kf(x_k)ge f(sum_{k=1}^{n}x_klambda_k)$.



The previous question was to demonstrate that ;
$zin ]x,y[ <=> existslambda, z=xlambda + (1-lambda)y$










share|cite|improve this question









$endgroup$




having a bit of trouble with this exercise;



Let f be a function, convex on $mathbb{I}$, by writing $sum_{k=1}^{n}x_klambda_k $ for $ n ge 3 $ under the form $x_nlambda_n+(1-lambda_n)y_n$ with $y_n$ expressed with $x_k, kin [|1,n|] $ and $ lambda_q, qin[|1,n|]$ and $lambda_iin]0,1[$and $sum_{n}^{i=1}lambda_i = 1$ .
Prove that :
$sum_{k=1}^{n}lambda_kf(x_k)ge f(sum_{k=1}^{n}x_klambda_k)$.



The previous question was to demonstrate that ;
$zin ]x,y[ <=> existslambda, z=xlambda + (1-lambda)y$







convex-analysis






share|cite|improve this question













share|cite|improve this question











share|cite|improve this question




share|cite|improve this question










asked Dec 25 '18 at 16:54









user310148user310148

43




43




closed as off-topic by amWhy, Paul Frost, Eevee Trainer, Saad, Lord_Farin Dec 26 '18 at 9:33


This question appears to be off-topic. The users who voted to close gave this specific reason:


  • "This question is missing context or other details: Please provide additional context, which ideally explains why the question is relevant to you and our community. Some forms of context include: background and motivation, relevant definitions, source, possible strategies, your current progress, why the question is interesting or important, etc." – amWhy, Paul Frost, Eevee Trainer, Saad, Lord_Farin

If this question can be reworded to fit the rules in the help center, please edit the question.







closed as off-topic by amWhy, Paul Frost, Eevee Trainer, Saad, Lord_Farin Dec 26 '18 at 9:33


This question appears to be off-topic. The users who voted to close gave this specific reason:


  • "This question is missing context or other details: Please provide additional context, which ideally explains why the question is relevant to you and our community. Some forms of context include: background and motivation, relevant definitions, source, possible strategies, your current progress, why the question is interesting or important, etc." – amWhy, Paul Frost, Eevee Trainer, Saad, Lord_Farin

If this question can be reworded to fit the rules in the help center, please edit the question.












  • $begingroup$
    May be n dimensional Jensen's inequality may fit the bill?
    $endgroup$
    – Satish Ramanathan
    Dec 25 '18 at 17:26






  • 1




    $begingroup$
    This needs rewriting. It is strangely confusing.
    $endgroup$
    – zhw.
    Dec 25 '18 at 17:59


















  • $begingroup$
    May be n dimensional Jensen's inequality may fit the bill?
    $endgroup$
    – Satish Ramanathan
    Dec 25 '18 at 17:26






  • 1




    $begingroup$
    This needs rewriting. It is strangely confusing.
    $endgroup$
    – zhw.
    Dec 25 '18 at 17:59
















$begingroup$
May be n dimensional Jensen's inequality may fit the bill?
$endgroup$
– Satish Ramanathan
Dec 25 '18 at 17:26




$begingroup$
May be n dimensional Jensen's inequality may fit the bill?
$endgroup$
– Satish Ramanathan
Dec 25 '18 at 17:26




1




1




$begingroup$
This needs rewriting. It is strangely confusing.
$endgroup$
– zhw.
Dec 25 '18 at 17:59




$begingroup$
This needs rewriting. It is strangely confusing.
$endgroup$
– zhw.
Dec 25 '18 at 17:59










1 Answer
1






active

oldest

votes


















2












$begingroup$

hint



$$sum_{i=1}^nlambda_ix_i=$$



$$lambda_nx_n+(1-lambda_n)sum_{i=1}^{n-1}frac{lambda_i x_i}{1-lambda_n}=$$



$$lambda_nx_n+(1-lambda_n)y_n$$






share|cite|improve this answer









$endgroup$













  • $begingroup$
    Hey thanks yea, sorry i forgot to say i got that part already sorry
    $endgroup$
    – user310148
    Dec 25 '18 at 18:50


















1 Answer
1






active

oldest

votes








1 Answer
1






active

oldest

votes









active

oldest

votes






active

oldest

votes









2












$begingroup$

hint



$$sum_{i=1}^nlambda_ix_i=$$



$$lambda_nx_n+(1-lambda_n)sum_{i=1}^{n-1}frac{lambda_i x_i}{1-lambda_n}=$$



$$lambda_nx_n+(1-lambda_n)y_n$$






share|cite|improve this answer









$endgroup$













  • $begingroup$
    Hey thanks yea, sorry i forgot to say i got that part already sorry
    $endgroup$
    – user310148
    Dec 25 '18 at 18:50
















2












$begingroup$

hint



$$sum_{i=1}^nlambda_ix_i=$$



$$lambda_nx_n+(1-lambda_n)sum_{i=1}^{n-1}frac{lambda_i x_i}{1-lambda_n}=$$



$$lambda_nx_n+(1-lambda_n)y_n$$






share|cite|improve this answer









$endgroup$













  • $begingroup$
    Hey thanks yea, sorry i forgot to say i got that part already sorry
    $endgroup$
    – user310148
    Dec 25 '18 at 18:50














2












2








2





$begingroup$

hint



$$sum_{i=1}^nlambda_ix_i=$$



$$lambda_nx_n+(1-lambda_n)sum_{i=1}^{n-1}frac{lambda_i x_i}{1-lambda_n}=$$



$$lambda_nx_n+(1-lambda_n)y_n$$






share|cite|improve this answer









$endgroup$



hint



$$sum_{i=1}^nlambda_ix_i=$$



$$lambda_nx_n+(1-lambda_n)sum_{i=1}^{n-1}frac{lambda_i x_i}{1-lambda_n}=$$



$$lambda_nx_n+(1-lambda_n)y_n$$







share|cite|improve this answer












share|cite|improve this answer



share|cite|improve this answer










answered Dec 25 '18 at 18:45









hamam_Abdallahhamam_Abdallah

38.1k21634




38.1k21634












  • $begingroup$
    Hey thanks yea, sorry i forgot to say i got that part already sorry
    $endgroup$
    – user310148
    Dec 25 '18 at 18:50


















  • $begingroup$
    Hey thanks yea, sorry i forgot to say i got that part already sorry
    $endgroup$
    – user310148
    Dec 25 '18 at 18:50
















$begingroup$
Hey thanks yea, sorry i forgot to say i got that part already sorry
$endgroup$
– user310148
Dec 25 '18 at 18:50




$begingroup$
Hey thanks yea, sorry i forgot to say i got that part already sorry
$endgroup$
– user310148
Dec 25 '18 at 18:50



Popular posts from this blog

Quarter-circle Tiles

build a pushdown automaton that recognizes the reverse language of a given pushdown automaton?

Mont Emei